right answer gets brainlist pls help asap

Right Answer Gets Brainlist Pls Help Asap

Answers

Answer 1
6 units

Explanation:
There are many ways. Easiest one is that u can literally count how many units between -7 and -1. Your answer is gonna be six units
Another way is by using taking the absolute value of their difference so you can say
|-1-(-7)| or |-7-(-1)| which simplifies to |6| and |-6| and this both are going to equal 6.
Answer 2

Answer:

6 units

Step-by-step explanation:

count the units between the two points


Related Questions

PLEASE HELP! due soon

Answers

Answer:

Your answer is 103.2413793103448

Step-by-step explanation:

[tex]29.94[/tex] ÷ [tex]29[/tex] [tex]= 1.032413793103448[/tex]

[tex]1.032413793103448[/tex] × [tex]100 = 103.2413793103448[/tex]

24.94 is 29% of 86
86 is your answer

Monica deposits $200 into a savings account that pays a simple interest rate of 4.4%. Paul deposits $300 into a savings account that pays a simple interest rate of 3.3%. Monica says that she will earn more interest in 1 year because her interest rate is higher. Is she correct? Justify your response.

Answers

Formula of simple interest rate:

[tex]S\mathrm{}I=\frac{P\times R\times T}{100}[/tex]

Where,

[tex]\begin{gathered} P=\text{ Principal} \\ R=\text{ Rate} \\ T=\text{ Time} \end{gathered}[/tex]

For Monica interest rate is:

[tex]\begin{gathered} S\mathrm{}I\mathrm{}=\frac{200\times4.4\times1}{100} \\ =8.8 \end{gathered}[/tex]

For Paul interest rate is:

[tex]\begin{gathered} S\mathrm{}I\mathrm{}=\frac{300\times3.3\times1}{100} \\ =3\times3.3 \\ =9.9 \end{gathered}[/tex]

No, Paul interest rate higher then Monic.

Jimmy’s family moved to a tropical climate. For the year that followed, he recorded the number of days that had a temperature above 40 degrees C each month. His data contained -
1-8 is for the data set below
14, 14, 10, 12, 11, 13, 11, 11, 14, 10, 13 and 8


1) Find the mean for his data set of days that had a temperature above 40 degrees C.

2) Find the median for his data set of days that had a temperature above 40 degrees C.

3) Find the mode for his data set of days that had a temperature above 40 degrees C.

4) If, instead, there are 5 more days per month that had a temperature above 40 degrees C, what will be the mean for the data?

5) If, instead, there are 2 more days per month that had a temperature above 40 degrees C, what will be the mode for the data?

6) If the number of days per month that had a temperature above 40 degrees C, doubles each month in that year, what will be the median for the data?

7) For what value of x will 9, 16 and x have the same mean (average) as that of 26 and 12?

8) For what value of x will 55 and x have the mean (average) as 67?

9) The mean (average) weight of three boys is 40 pounds. One of the boys weighs 50 pounds. The other two boys have the same weight. Find weight of each of the boys?

10) A cat consumes 2 cups of milk every day. How much milk does that cat drink on an average in a week?

11) What is mode for above question?

Answers

The mean, median and mode are as follows:

1. 11.75

2. 11.5

3. 11 and 14

4. 16.75

5. 13 and 16

6. 23

7. 32

8.  79

9.  35

10.  14

11.  2

What are mean, median and mode ?

The three measures of central tendency used in statistics are mean, median, and mode. While describing a set of data, we always specify the focal point of any data set. The measure of central tendency is this. Every day, data are presented to us. We discover them in books, articles, bank statements, phone and utility bills, as well as in newspapers. There are so many; they are all around us. The challenge now is if we can identify some key characteristics of the data by focusing on only a few data points. The means, medians, and modes—measures of central tendency or averages—can be used to do this. When describing a set of data, a measure of central tendency locates the set's center.

To know more about mean median and mode ,visit:

brainly.com/question/14859109

#SPJ13

Write the factored form of the polynomial function with real coefficients, a lead coefficient of 1, and zeros of − 2 , − 4 , 3 , and 3 .

Answers

The equation of the polynomial equation is P(x) = (x + 2)(x + 4)(x - 3)(x + 3)

How to determine the polynomial equation?

The given parameters are

Leading coefficient = 1

Zeros =  − 2 , − 4 , 3 , and 3 .

Polynomial equations have their multiplicities to add up to their degree.

This means that the multiplicity of the zeros is 1

The equation of the polynomial is then calculated as

P(x) = (x - zero)^ multiplicity

So, we have

P(x) = (x + 2)(x + 4)(x - 3)(x + 3)

From the question, the polynomial is to be factored

This means that we represent it as

P(x) = (x + 2)(x + 4)(x - 3)(x + 3)

Hence, the polynomial equation is P(x) = (x + 2)(x + 4)(x - 3)(x + 3)

Read more about polynomial at

brainly.com/question/17517586

#SPJ1

Polynomial is an expression consisting of indeterminates and coefficients the factored form of polynomial function is (x+2)(x+4)(x-3)(x-3)

What is Polynomial?

Polynomial is an expression consisting of indeterminates and coefficients, which involves  the operations like addition, subtraction, multiplication, and positive-integer powers of variables

Factors are the numbers which divide the expression or number with a remainder zero.

We need to write the factored form of the polynomial functions with real; coefficients and zeros minus two comma  minus four comma three and 3.

The leading coefficient of the polynomial is 1. Leading coefficient means coefficient of leading term.

Zeros =  − 2 , − 4 , 3 , and 3 .

Let the polynomial is of variable x.

The equation of the polynomial is then calculated as

P(x) = (x - zero)^ multiplicity

(x-(-2))=x+2

(x-(-4))=x+4

(x-3)=x-3

So the factored form is (x+2)(x+4)(x-3)(x-3)

the polynomial is P(x)=(x+2)(x+4)(x-3)(x-3)

Hence the factored form of polynomial function is (x+2)(x+4)(x-3)(x-3).

To learn more on polynomial click:

https://brainly.com/question/11536910

#SPJ1

s\\line t find the measure of each angle HELP ME NOWWW

Answers

The values of the angles for the two parallel lines will be ∠1 = 115°, ∠2=115°, ∠3=148°, and ∠4 = 148°.

What are lines and angles?

Straight lines with little depth or width are present. You will learn about a number of lines, including transversal, intersecting, and perpendicular lines. A figure called an angle is one in which two rays originate from the same point.

Given that the lines s and t are parallel to each other. The values of the different angles will be:-

∠1  = 115° Exterior angle property

∠2 = 115° Vertically opposite angle

The value of angle ∠3 will be calculated by the supplementary angle property.

∠3 = 148°

The value of angle ∠4 is 148 by vertically opposite angle property.

∠4 = 148°

To know more about lines and angles follow

https://brainly.in/question/2710053

#SPJ1

What is the answer to the following calculation, rounded to the correct number of significant figures?100.000 g+ 75.0 g

Answers

Answer:

175g

Step-by-step explanation:

100.000g+75.0g= 175g

ps. pls give brainliest answer :)

The sum of the numbers 100.000g and 75.0g in expression is 175g.

What are mathematical operations?

Calculate the answer using a math operator is referred to as a mathematical operation.

Basic mathematical operations are addition, multiplication, subtraction and division.

The given numbers are,

100.000g and 75.0g

The zeros can be neglected after decimal points,

So the numbers can be written as,

100g and 75g.

To find the required expression, add 100g and 75g.

100g + 75g = 175g.

The required sum of the numbers is 175g.

To learn more about Mathematical operations on :

https://brainly.com/question/22469627

#SPJ2

six distinct positive integers are randomly chosen between and , inclusive. what is the probability that some pair of these integers has a difference that is a multiple of ?

Answers

Answer:

So the probability of this happening is exactly 1 - it must be true

Step-by-step explanation:

If we calculate modulo(5) for each of the six numbers (the integer remainder after dividing by 5), we will get six values from 0 to 4.

By the pigeonhole principle, since there are 6 numbers and only 5 possible values, at least two must share the same value modulo 5. Pick two of those, say x and y, in which case x mod 5 = y mod 5, therefore (x - y) mod 5 = 0. In other words, their difference is a multiple of 5.

So the probability of this happening is exactly 1 - it must be true

At a local print shop, 10 copies can be made for $4. At this rate, how much would it cost to make 75 copies?​

Answers

Answer:

$30

Step-by-step explanation:

10 x 7 = 70

7 x 4 = 28

Since it's $4 to make 10 copies, it'll be $2 to make 5

28 + 2 = 30

$30 for 75 copies (a bad deal honestly)

Please mark Brainliest!!

Answer:

$30 dollars for 75 copies

Step-by-step explanation:

What we know:

$4 for every 10 copies.

How to figure out:

Divide 4/10 = .4    .4 is the cost per copy.

We do this because if it is 4 dollars per 10 copies, then we have to figure out the cost per copy. So we have to do how many dollars goes into each copy.

Now we know the cost of one copy we can multiply the amount of copies we need by the price of one.

75 x 0.4 = 30     30 is the price of 75 copies.

What percentage of people in the study started
saving for retirement at age 21 or before? How
does this compare with the how many of these
same people believe their children or
grandchildren should start saving at 21 or
before?

Answers

Answer:8% started saving at or before 21

30% believe their children or grandchildren should start saving at or before 21

Step-by-step explanation: on the graph, the ones in lighter color start saving for requirements

That is 3%+5%=8%

The second solution goes thus

13%+17%= 30%

the ones in dark colors believe their children or grandchildren should start saving before 21

Elijah put 2x+3
2
x
+
3
dollars in the bank the first week. The following week he doubled the first week’s savings and put that amount in the bank. The next week, he doubled what was in the bank and put that amount in the bank. He now has $477 in the bank. How much money did he put in the bank the first week?

Answers

Elijah put $53 in the Bank in the first week .

In the question ,

it is given that Elijah put (2x+3) in the first week .

In the second week Elijah put double the first week savings

that means second week deposit = 2*(2x+3) = 4x+6

in the third week he doubled the amount that was in the bank ,

which means third week deposit = 2*(first week + second week deposit)

= 2*(2x+3+4x+6)

Also given that total amount in the bank = $477

total amount = first week + second week + third week deposit

substituting the values we get

477 = (2x+3) + (4x+6) + 2*(2x+3+4x+6)

477 = 2x+3+4x+6+4x+6+8x+12

477 = 18x + 27

18x = 477-27

18x = 450

x = 450/18

x = 25

the amount deposited in the first week = 2x+3

                                                                  = 2(25)+3

                                                                  = 50+3 = $53

Therefore , Elijah put $53 in the Bank in the first week .

The given question is incomplete , the complete question is

Elijah put 2x+3 dollars in the bank the first week. The following week he doubled the first week’s savings and put that amount in the bank. The next week, he doubled what was in the bank and put that amount in the bank. He now has $477 in the bank. How much money did he put in the bank the first week?

Learn more about Equation here

https://brainly.com/question/14623146

#SPJ1

Emily estimated that there were
315 students at a soccer game.
The actual number of students at
the game was 350. What was the
percent error of her estimate?


I thought it was 8%. Please show your work, and maybe tell me why I was right or wrong? Thank you!

Answers

The percentage error in the measurement of the number of student at the soccer game. given that 315 student was estimated and 350 student were actually at the game is 10%

What is a percentage error?

The percentage error is the measure of the difference between an actual value and a estimate given as a percentage.

The percentage error of her estimate is found from the percentage error formula, as follows;

[tex]\delta = \dfrac{v_A - v_E}{v_E} \times 100\%[/tex]

Where:

δv=The percentage error

[tex]v_A[/tex] = The approximate value or estimated value = 315

[tex]v_E[/tex] = The exact or correct value = 350

Therefore; [tex]\delta = \dfrac{|315 - 350|}{350} \times 100\%= 10\%[/tex]

The percentage error of her estimate is 10%

The method and information used in calculating the percentage error determines the value of the error in percentage, however, based on the values of estimated value of 315, and the actual value of 350, the percentage error is 10%

Learn more about percentage error of a measurement here:

https://brainly.com/question/5493941

#SPJ1

Can u please help me with this answer

Answers

Answer:

option a as x=8 hope you understand my writing , if got some problem comment down

1) If a horizontal asymptote exists for this function, identify its location.4x + 6x3x3 - 2x + 1AyoB3B) y =4OyD Does Not Exist

Answers

For this problem, we are given the following rational function:

[tex]f(x)=\frac{4x^3+6x}{3x^3-2x+1}[/tex]

We need to determine the horizontal asymptote for this function. In order to determine this, we need to calculate the limit of the function when x approaches infinity. We have:

[tex]\lim_{x\rightarrow\infty}\frac{4(\infty)^3+6\cdot\infty}{3(\infty)^3-2\cdot\infty+1}=\frac{4}{3}[/tex]

The horizontal asymptote exists at y= 4/3. The correct option is C.

2/3 is what percent of 1/4? of 1/6

Answers

Let's first calculate the percentage for 1/4

[tex]\begin{gathered} P=\frac{2/3}{1/4} \\ P=\frac{8}{3} \\ P=2.666 \\ P=267\text{ \%} \end{gathered}[/tex]

Let's first calculate the percentage for 1/6

[tex]\begin{gathered} P=\frac{2/3}{1/6} \\ P=\frac{12}{3} \\ P=4 \\ P=400\text{ \%} \end{gathered}[/tex]

Find the equation to the line below.y =[?-x ++ [ ]

Answers

Note that a line whose equation is in the form :

[tex]y=mx+b[/tex]

has a slope of m

and a y-intercept of b

Slope is also equal to rise/run :

[tex]m=\frac{\text{rise}}{\text{run}}[/tex]

where rise is the number of vertical units between two points and run is the number of horizontal units between two points.

Note also that upward and right movement is positive, downward and left movement is negative.

From the figure, the vertical units between two given points is 3 units upward, so this is positive.

The horizontal units between them is 2 units to the right which is also positive.

The slope will be :

[tex]m=\frac{\text{rise}}{\text{run}}=\frac{3}{2}[/tex]

Next is to solve for b (y intercept).

y-intercept is the point where the line intersects the y-axis.

From the graph, the lines intersects at point (0, 2) on the y-axis

Therefore, the value of b is 2

Rewriting the equation, the answer is :

[tex]\begin{gathered} y=mx+b \\ y=\frac{3}{2}x+2 \end{gathered}[/tex]

PLSSSSS
WILL MARK BRAINLIEST!!!
SIMPLEEE
(Yr7)

Answers

Answer:

Step-by-step explanation:

the answer I have given below

kind regards

For Point A (m,n) to fall in the correct quadrant, if the value of m is 6, the 6 would be
, and if the value of n is 3, the 3 would be

6 positive or negative
3 positive or negative

Here's the picture if you are confused I guess

Answers

6 must be negative and 3 is also negative

Which angle pair is NOT an example of Corresponding Angles?

angle 2 and angle 10

angle 1 and angle 11

angle 13 and angle 15

angle 8 and angle 16​

Answers

Answer:

Step-by-step explanation:  

So briefly corresponding angles are angles that have the same position as another angle on the same figure. In your example there a multiple figures. The only pair that is NOT a corresponding angle is 1 and 11 because they have no lines in common they are not on the same figure! Hope this helps!

Given f(x) = -x - 2, find f(-6).

Answers

Answer:

f( -6) = 4

Step-by-step explanation:

f(x) = -x -2  You will replace x with -6

f(-6) - -(-6) -2

f(-6) = 6 - 2

f(-6) = 4

Which operation comes first in the problem below? 4(3.7 +1.23) -12 A. Addition B. Subtraction C. Multiplication D. Division

Answers

A. Addition

Explanation

PEMDAS is an acronym for the words parenthesis, exponents, multiplication, division, addition, subtraction. Given two or more operations in a single expression, the order of the letters in PEMDAS tells you what to calculate first, second, third and so on, until the calculation is complete

Step 1

so, firts calculate the parenthesis

[tex]\begin{gathered} 4(3.7+1.23)-12 \\ 4(4.93)-12 \end{gathered}[/tex]

the operation in parenthesis is an addition

i hope this helps you

WILL GIVE BRAINLOST TO BEST QUSTION

The cost of a ride in a taxi is $4 for the first half-mile plus a constant amount per half-mile after that. The sequence below shows the pattern of numbers that appear on the driver's screen.

2.50, 2.75, 3, 3.25, 3.50,...

Which of the following is the Recursive Function that can be used to determine f(x), the cost in dollars, for a ride in the tax of, x. half miles?

Answers

Answer:

I don't know men iam grade 7 student

pls explain with working out

Answers

Answer: 26m

Step-by-step explanation:

The area is 36m^2, the possible measurement that could've worked is 4m for width and 9m for length (4x9=36). Now just do 4+4+9+9 = 26m.

Hope this helped

A shipping box has a volume of 1,000 cubic inches. What is the length of one side of the box?

Answers

The length of one side of the box is 10 inches.

How to find the volume of a cube?

Suppose that: The side length of the considered cube is L units.

Then, we get the Volume of that cube = L³ cubic units.

The volume of a cube is of the form V =  L³, where L is the side length.

Given that the shipping box has a volume of 1,000 cubic inches.

Here, we know the volume V, but not the measure of length. So, we can plug in the value 1000 for V and solve for L:

1000 =  L³

Cube root both sides:

L = [tex]\sqrt[3]{1000}[/tex]

Therefore, each side is 10 inches.

Learn more about the volume of a cube here:

https://brainly.com/question/26136041

#SPJ1

5. How many ways are there to distribute 10 indistinguishable candies among 4 different
children? Children may end up with no candies.
PLSSSSS HELP IT IS EXTREMELY URGENT PLSSSS

Answers

By application of the combination formula, there are 210 ways for distributing 10 indistinguishable candies among 4 children.

What is combination?

Combination is the arrangement of objects in which order is not taken into account.

The applicable formula is:

n combination r = n!/[(n - r)!r!]

where n is the number of indistinguishable items (10 candies), and r is the possible number of recipients (4 kids).

Hence;

10 combination 4 = 10!/[(10 - 4)!4!] ways

10 combination 4 = 10!/(6! × 4!) ways

10 combination 4 = (10 × 9 × 8 × 7 × 6!)/(6! × 4 × 3 × 2 × 1) ways

10 combination 4 = 10 × 3 × 7 ways

10 combination 4 = 210 ways

Therefore, there are 210 ways for the 10 indistinguishable candies to be distributed among the 4 children by with the application of combination formula.

Learn more about combination here: https://brainly.com/question/11732255

#SPJ1

Angle P in Triangle PQR has the same measure as Angle S in Triangle STU. Which other condition is necessary to prove that these triangles are similar?

Answers

△PQR is congruent to △STU (△PQR ≅ △STU) under ASA condition using options (B) and (D).

What is the congruency of triangles?Triangle congruence: If all three corresponding sides and all three corresponding angles are equal in size, two triangles are said to be congruent. Slide, rotate, flip, and turn these triangles to create an identical appearance. They are in alignment with one another when moved. Therefore, if all three sides of two triangles are the same, then the triangles are said to be congruent. If we have a side, an angle between the sides, and then another side that is congruent, we know they are congruent. In other words, side, angle, side.

So, to prove that △PQR ≅ △STU:

∠P = ∠S (Given)PQ = ST (Option B)∠Q = ∠T (Option D)

So, with these three conditions, △PQR ≅ △STU is under ASA condition.

Therefore, △PQR is congruent to △STU (△PQR ≅ △STU) under ASA condition using options (B) and (D).

Know more about the congruency of triangles here:

https://brainly.com/question/2938476

#SPJ13

A guy wire runs from the top of a cell tower to a metal stake in the ground. Hannah
places a 6-foot tall pole to support the guy wire. After placing the pole, Hannah
measures the distance from the stake to the pole to be 1 ft. She then measures the
distance from the pole to the tower to be 12 ft. Find the length of the guy wire, to the
nearest foot.

Answers

The length of the guy wire, to the nearest foot is 1548 ft.

What is the Pythagoras theorem?

The right-angled triangle's relationship between its three sides is explained by the Pythagoras theorem, commonly known as the Pythagorean theorem. The square of a triangle's hypotenuse is equal to the sum of its other two sides' squares, according to the Pythagoras theorem. The hypotenuse's square is equal to the sum of the squares of the other two sides if a triangle has a straight angle (90 degrees), according to the Pythagoras theorem. Keep in mind that BC² = AB² + AC² in the triangle ABC signifies this. Base AB, height AC, and hypotenuse BC are all used in this equation. The longest side of a right-angled triangle is its hypotenuse, it should be emphasized

After sketching the information given, we have two similar right triangles, ΔABE and ΔCDE.

CD = 6 ft

DE = 1 ft

BD = 12 ft

Since ΔABE ~ ΔCDE, therefore,

AB/CD = BE/DE (proportional sides)

Plug in the values

AB/6 = (12 + 1)/2

AB/6 = 13/2

Cross multiply

AB = 13(3)

AB = 39 ft

AE = √(AB² + BE²)

AE = √(39² + 12²)

AE = 1548 ft (nearest foot)

To know more about Pythagoras theorem ,visit:

brainly.com/question/343682

#SPJ13

Answer: The length of the guy wire, to the nearest foot is 1548 ft.

Step-by-step explanation:

what are the chances that radio transmissions from earth or messages sent on distant space probes will ever be received by living beings? what are the chances that radio transmissions from earth or messages sent on distant space probes will ever be received by living beings? extremely high. very low. about 50%.

Answers

The chances that radio transmissions from Earth or messages sent on distant space probes will ever be received by living beings are very low.

What is meant by radio transmission?

In radio transmission, a radio transmitter's antenna couples electromagnetic energy into the atmosphere or other open space. Electromagnetic radio waves are caught by a receiving antenna during radio reception and coupled into a receiver for detection.

Radio waves have a low frequency, which limits their ability to deliver large amounts of data at once.

There are extremely slim odds that radio signals from Earth or messages placed on far-off space probes will ever be received by living things, it is acceptable to the state. Remember that some scientists have held this belief about the existence of extraterrestrial life for years without any convincing evidence.

To learn more about radio transmission refer to:

https://brainly.com/question/13033832

#SPJ4

can you help solve for y?​

Answers

Answer:

Go ahead and color the nose red!

The solution is
[tex]y = \dfrac{n-9}{x}[/tex]

Step-by-step explanation:

[tex]\text{We have: }\\\\[/tex]

[tex]\rightarrow\; \dfrac{x}{y} + 9 = n\\\\\\\text{Subtract 9 from both sides:}\\\\\rightarrow\;\dfrac{x}{y} + 9 - 9 = n - 9\\\\[/tex]

[tex]\rightarrow\;\dfrac{x}{y} = n - 9\\\\[/tex]

Multiply both sides by y:

[tex]\rightarrow\; y \cdot \dfrac{x}{y} = y (n-9)\\\\\rightarrow\; x = y(n-9)\\\\\\[/tex]

[tex]\text{Divide both sides by (n-9)}:\\\\\\\rightarrow\; \dfrac{x}{n-9} = \dfrac{y (n-9)}{n-9}\\\\\\\rightarrow\; \dfrac{x}{n-9} = y\\\\\\[/tex]

By switching sides without changing the meaning, this can be re-written as :

[tex]y = \dfrac{x}{n-9} \\\\[/tex]

which of course means you can color the nose red

:)

provide an appropriate response. the owner of a computer repair shop has determined that their daily revenue has mean $7200 and standard deviation $1200. the daily revenue totals for the next 30 days will be monitored. what is the probability that the mean daily revenue for the next 30 days will be less than $7000?

Answers

The probability that the mean daily revenue for the next 30 days will be less than $7000 IS 0.7333

Standard deviation of the sample is equal to the population's Standard Deviation divided by the square root of the sample's item count.

Standard Deviation of the sample = [tex]1200/\sqrt{30} = 219.09[/tex]

SD of the sample is equal to the population's Standard Deviation divided by the square root of the sample's item count.

To reach a result of 300, divide 7500 by the mean of 7200. 300 divided by the sample's SD (219.09) yields a result of 1.37. A z-table search for +1.37 returns a result of 0.9147. Accordingly, there is a 0.9147 percent chance that the average daily revenue over the following 30 days will be less than $7500.

To get a result of -200, divide 7000 by the mean of 7200. Divide -200 by the sample's standard deviation (219.09), and you'll get -0.91. search for -0.91 in a z-table receive a score of 0.1814. The likelihood that the mean daily revenue for the following 30 days would be less than $7000 is therefore 0.1814.

Simply subtract the likelihood that the mean daily revenue is less than $7000 (0.1814) from the likelihood that the mean daily revenue is less than $7500 (0.9147) to arrive at the probability that the mean daily revenue for the upcoming 30 days will be between $7000 and $7500. This yields a result of 0.7333.

There is a 73.33% chance that the average daily income over the following 30 days will be less than $7000.

To learn more about Standard Deviation:

https://brainly.com/question/13498201

#SPJ4

How many students age 15 and above take a car to school? Show work
A. 18
B. 38
C. 87

Answers

im pretty sure its B or most likeley C

b:38

98 (total amt of students who take cars) - 60 (amount of students under 15 who take cars) = 38
Other Questions
you are driving a 30 foot bus on a highway at 45 mph. the road is dry and visibility is good. a safe distance between you and the vehicle ahead of you should be at least: q divided by 4 + 8q, for q=8 what is the least common denominator for the two fractions 2 / 5 3 / 2 nominal variables are variables that are measured in:multiple choice question.employment units while real variables are measured by money.units of output while real variables are measured in prices.monetary units while real variables are measured in numerical units.numerical units while real variables are measured in monetary units. The circumference of a circle is 278.71m. What is the approximate area of the circle? Use 3.14 for pi. Explain how the area of a circle changes when the circumference of a circle changes ( round the final answer to the nearest whole number as needed , round all the intermediate values to the nearest thousandth as needed ) A 12-ft-by-15-ft rectangular swimming pool has a 3-ft-wide no-slip surface around it. What is the outer perimeter of the no-slip surface?. For which type of lesion would a bandage be most important in order to keep bacteria and other pathogens from entering the body?. Hello. I would like help with problem. Quick answer is OK.Thank you use the listing method to represent the following set. picture attached Could I assistance receive some on this question its very confusing a uniformly charged conducting sphere of 1.22 m radius has a surface charge density of 8.13 c/m*. (a) find the charge on the sphere. (4) what ts the total electric flux leaving the surface of the sphere? (c) calculate the electric field at the surface of the sphere. When there is civil unrest, is war inevitable? Theme: EnvironmentEssential Question: How did the environment impact the development of the different colonial economies? Qu factores contribuyeron a la formacin de la cultura china? A stock is worth $28,775 and drops 33% in one day. What percent does the stock have to grow the next day to get back to $28,775 A wave traveling on a Slinky that is stretched to 4 m takes 4.97 s to travel the length of the Slinky and back again.(a) What is the speed (in m/s) of the wave? m/s Find the values of sin 0, cos 0, and tan e for the given right triangle. Give the exact values.sin 0=cos 0=tan 0=87 A lighthouse beacon will illuminate to a distance of 12 km. If the lighthouse is located at (-5,2) on a grid, find the equation of the location of the furthest points lit the beacon. a sample of a material has 20002000 radioactive particles in it today. your grandfather measured 40004000 radioactive particles in it 6060 years ago. how many radioactive particles will the sample have 60 years60 years from today? 48. In the parabola, y = 3x ^ 2 + 12x + 11 focus is located at a distance p > 0 from the vertex. Then p=a. 3b. 1/3c. 12d. 1/12e. None of the above